Difference between revisions of "2021 AMC 10A Problems/Problem 11"
(→Solution 2) |
(→Solution 1) |
||
Line 8: | Line 8: | ||
~MRENTHUSIASM | ~MRENTHUSIASM | ||
− | |||
==Solution 2== | ==Solution 2== |
Revision as of 13:29, 5 May 2021
Contents
Problem
For which of the following integers is the base- number not divisible by ?
Solution 1
We have This expression is divisible by unless The only choice congruent to modulo is
~MRENTHUSIASM
Solution 2
Vertically subtracting 221_b, we see that the ones place becomes 0, the b^1 place becomes 0 as well. Now, at the b^2 place, we must perform a carry, but instead of incrementing the place's value by 10 like we normally would in base 10, we do so by b, and make the b^3 place in 1100_b.
Video Solution (Simple and Quick)
~ Education, the Study of Everything
Video Solution
https://www.youtube.com/watch?v=XBfRVYx64dA&list=PLexHyfQ8DMuKqltG3cHT7Di4jhVl6L4YJ&index=10
~North America Math Contest Go Go Go
Video Solution 3
~savannahsolver
Video Solution by TheBeautyofMath
~IceMatrix
See Also
2021 AMC 10A (Problems • Answer Key • Resources) | ||
Preceded by Problem 10 |
Followed by Problem 12 | |
1 • 2 • 3 • 4 • 5 • 6 • 7 • 8 • 9 • 10 • 11 • 12 • 13 • 14 • 15 • 16 • 17 • 18 • 19 • 20 • 21 • 22 • 23 • 24 • 25 | ||
All AMC 10 Problems and Solutions |
The problems on this page are copyrighted by the Mathematical Association of America's American Mathematics Competitions.